Đến nội dung

Minhnguyenthe333 nội dung

Có 788 mục bởi Minhnguyenthe333 (Tìm giới hạn từ 21-04-2020)



Sắp theo                Sắp xếp  

#574661 Xác định k thuộc $\mathbb{R}$ để góc giữa $(...

Đã gửi bởi Minhnguyenthe333 on 22-07-2015 - 19:12 trong Hình học phẳng

[quote name="Wendy Sayuri" post="574648" timestamp="1437558411"]Cho 2 vectơ $\vec{a}, \vec{b}$ thoả mãn: $\left | \vec{a} \right |=1,\left | \vec{b} \right |=2, \left | \vec{a} - 2\vec{b} \right |=\sqrt{15}$
Xác định k thuộc $\mathbb{R}$ để góc giữa $(\vec{a}+\vec{b}), (2k\vec{a}-\vec{b})$ bằng 600.[/quote
Ta có:$\left | \vec{a} - 2\vec{b} \right |^2=15\Leftrightarrow \vec{a}.\vec{b}=\frac{1}{2}$
$\left | \vec{a} + \vec{b} \right |=\sqrt{6}$
$|2k\vec{a}-\vec{b}|=\sqrt{4k^2-2k+4}$
$( \vec{a} + \vec{b} )(2k \vec{a} -\vec{b})=3k-\frac{9}{2}$
Lại có:$cos60^0=\frac{1}{2}=\frac{( \vec{a} + \vec{b} )(2k \vec{a} -\vec{b})}{| \vec{a} + \vec{b} |.| 2k \vec{a} -\vec{b} |}$
$\Leftrightarrow 6k-9=\sqrt{24k^2-12k+24}$
$\Rightarrow k=...$



#609130 x, y là các số thực dương thỏa mãn: x + y = 1

Đã gửi bởi Minhnguyenthe333 on 15-01-2016 - 19:51 trong Bất đẳng thức và cực trị

Tìm giá trị lớn nhất, nhỏ nhất:

1)$B=\frac{1}{1-3xy}+\frac{1}{xy}=\frac{1}{1-3u}+\frac{1}{u}$
$<=>B=\frac{1-2u}{u(1-3u)}<=>3Bu^2-(B+2)u+1=0$
$\Delta=(B+2)^2-12B\geqslant 0<=>B\geqslant 4+2\sqrt{3}$



#612403 Viết PT $(\Delta)$ qua $M(4;1)$ cắt $Ox, Oy...

Đã gửi bởi Minhnguyenthe333 on 02-02-2016 - 11:48 trong Phương pháp tọa độ trong mặt phẳng

Giúp em với ạ tập đính kèm ở dưới thanks mọi người

Bài 1: Cho $\Delta ABC$ có $A(-1;-3)$ đường trung trực của $AB : 3x + 2y – 4 = 0, G( 4;-2)$ là trọng tâm $\Delta ABC$
Viết phương trình cạnh $BC$. Tìm $B, C$
Bài 2: Viết phương trình đường thẳng $(\Delta)$ qua $M(4;1)$ cắt $Ox, Oy$ tại
$A, B$ theo các trường hợp sau:
a). Diện tích $\Delta OAB$ nhỏ nhất
b). $OA + OB$ nhỏ nhất


p/s: ghi lên đây cho mọi người dễ làm

2/
a)Ta có $OA>4,OB>1$ nên ta đặt $u,v$ sao cho $OA=v+4;OB+u+1$ và $Ox_1=4;Oy_1=1$ (giả thiết)
$S=_{OAB}=OA.OB.\frac{1}{2}=(v+4)(u+1).\frac{1}{2}$
$\Delta By_1M \sim \Delta Mx_1A=>\frac{u}{4}=\frac{1}{v}<=>u=\frac{4}{v}$
$=>(v+4)(u+1)=(v+4)(\frac{4}{v}+1)<=>\frac{S_{OAB}}{v}=\frac{1}{2}(\frac{4}{v}+1)^2\geqslant \frac{8}{v}$
$=>S_{OAB}\geqslant 8$.Dấu "=" xảy ra khi $v=4;u=1<=>OA=8;OB=2$
$=>y=\frac{1}{4}x$

b)$S=OA+OB=v+u+5=\frac{v^2+5v+4}{v}<=>v^2+(5-S)v+4=0$
$\Delta \geqslant 0<=>S\geqslant 9$ hay $OA+OB\geqslant 9$
Dấu "=" xảy ra khi $v=2;u=2<=>OA=6;OB=3$
$=>y=x-3$
P/S: em mới học cấp 2 nên sai thì thông cảm :)



#587623 Tuyển tập các bài khó trong kì thi chuyên,HSG lớp 9 các năm

Đã gửi bởi Minhnguyenthe333 on 06-09-2015 - 15:14 trong Bất đẳng thức và cực trị

Bài 1: Cho $n\in \mathbb{Z^+}$ và kí hiệu $U(n)={d_1,d_2,...,d_m}$ là tập hợp các ước nguyên dương của $n$.

Chứng minh rằng: $d_1^2+d_2^2+...+d_m^2\leq n^2\sqrt{n}$

 

Bài 2: Cho $a.b.c>0$.Chứng minh:

$\frac{a^2}{b}+\frac{b^2}{c}+\frac{c^2}{a}\geq \sqrt{a^2-ab+b^2}+\sqrt{b^2-bc+c^2}+\sqrt{c^2-ac+a^2}$

 

Bài 3: Cho $x,y,z\in (0;1)$.Chứng minh:

$\frac{1}{x(1-y)}+\frac{1}{y(1-z)}+\frac{1}{z(1-x)}\geq \frac{3}{xyz+(1-x)(1-y)(1-z)}$

 

Bài 4: Tìm giá trị của $k$ để phương trình sau có nghiệm

             $(x^2+2)[x^2-2x(2k-1)+5k^2-6k+3]=2x+1$

 

Bài 5: Giải phương trình: $x+1=\sqrt{2(x+1)+2\sqrt{2(x+1)+2\sqrt{4(x+1)}}}$

 

Bài 6: Giải bất phương trình: $\sqrt{x^4+x^2+1}+\sqrt{x(x^2-x+1)}\leq \sqrt{\frac{(x^2+1)^3}{x}}$

 

Bài 7: Cho số tự nhiên $n>1$ và $n+2$ số nguyên dương $a_1,a_2,...,a_{n+2}$ thỏa $1\leq a_1\leq...\leq a_{n+2}\leq 3n$

Chứng minh rằng tồn tại hai số $a_i,a_j(1\leq j\leq i\leq n+2)$ sao cho $n<a_i-a_j<2n$




#591033 Tuyển tập các bài bất đẳng thức khó trong kì thi HSG

Đã gửi bởi Minhnguyenthe333 on 26-09-2015 - 21:37 trong Bất đẳng thức và cực trị

Bài 1: Cho $a,b,c>0$ thỏa $a+b+c=3$.Chúng minh rằng:

               $\frac{a}{1+b^2}+\frac{b}{1+c^2}+\frac{c}{1+a^2}\geq \frac{3}{2}$

 

Bài 2: Cho $a,b,c>0$ thỏa: $\frac{1}{1+a+b}+\frac{1}{1+b+c}+\frac{1}{1+c+a}\geq 1$

 Chứng minh rằng: $a+b+c\geq ab+bc+ca$

 

Bài 3: Cho $x_1,x_2,...,x_n>0$ thỏa $x_1+x_2+...+x_n=k$

Tìm $GTNN,GTLN$ của biểu thức $P=x_1x_2...x_n$

 

Bài 4: Cho $n$ là số tự nhiên lớn hơn $1$.Gọi $m$ là trung bình cộng của tất cả các ước số của $n$.

 Chứng minh rằng: $\sqrt{n}\leq m\leq \frac{n+1}{2}$

 

Bài 5: Cho $a,b>0$.Tìm hằng số $k$ lớn nhất thỏa:

  $\frac{k}{a^3+b^3}+\frac{1}{a^3}+\frac{1}{b^3}\geq \frac{16+4k}{(a+b)^3}$

 

Bài 6: Cho $a,b,c>0$ thỏa $abc=1$.Chứng minh\

               $\frac{1}{\sqrt{1+8a}}+\frac{1}{\sqrt{1+8b}}+\frac{1}{\sqrt{1+8c}}\geq 1$




#591416 Tuyển tập các bài bất đẳng thức khó trong kì thi HSG

Đã gửi bởi Minhnguyenthe333 on 30-09-2015 - 21:27 trong Bất đẳng thức và cực trị

Thêm vài bài:

Bài 7: Cho $a,b,c$ thỏa $a\geq b\geq c>a-b$ và $a+b+c=2m$.Chứng minh rằng:

            $[m(a+b-c)-ab][m(b+c-a)-bc][m(c+a-b)-ca]\leq \frac{a^2b^2c^2}{8}$

Bài 8: Tìm $GTNN,GTLN$ của biểu thức $f=\frac{mn}{mu^2+nv^2}$ trong đó $m,n,u,v\in \mathbb{Z^+}$ thỏa:

                         $u+v=20$ và $m+n=10$

Bài 9: Tìm giá trị nguyên lớn nhất của $m$ sao cho bất đẳng thức luôn đúng với mọi $x\in \mathbb{R}$:

                              $(x+1)(x+2)^2(x+3)\geq m$




#647440 Trại hè Hùng Vương 2016 Toán 10

Đã gửi bởi Minhnguyenthe333 on 31-07-2016 - 23:56 trong Thi HSG cấp Tỉnh, Thành phố. Olympic 30-4. Đề thi và kiểm tra đội tuyển các cấp.

Bài 3:
Đổi biến $\left ( \frac{a}{a+b},\frac{b}{b+c},\frac{c}{c+a} \right )\rightarrow (1-x,1-y,1-z)$
Dễ thấy $xyz=(1-x)(1-y)(1-z)\iff 2xyz=1+\sum x-\sum xy$
BĐT$\iff \sum (1-x)^2+4xyz\geqslant 3-(x+y+z)+\frac{1}{4}$
$\iff x^2+y^2+z^2+4xyz\geqslant x+y+z-\frac{1}{4}$
Thay $2xyz=1+\sum x-\sum xy$
$\iff x^2+y^2+z^2+2+2(x+y+z)-2(xy+yz+zx)\geqslant x+y+z-\frac{1}{4}$
$\iff (x+y+z)^2-3(x+y+z)+\frac{9}{4}\geqslant 0\iff [2(x+y+z)-3]^2\geqslant 0$ (luôn đúng)
Dấu "=" xảy ra khi $\sum \frac{a}{a+b}=\frac{3}{2}\iff ....$



#647600 Trại hè Hùng Vương 2016 Toán 10

Đã gửi bởi Minhnguyenthe333 on 02-08-2016 - 09:36 trong Thi HSG cấp Tỉnh, Thành phố. Olympic 30-4. Đề thi và kiểm tra đội tuyển các cấp.

Bài 5: Tìm các số $p,n $ thoả $p$ nguyên tố, $n$ là số nguyên dương sao cho

$p^3 -2p^2 + p +1= 3^n $

Giải theo kiểu này:

$\iff (p+1)(p^2-3p+4)=3(3^{n-1}+1)$

Đặt $d=\gcd(p+1,p^2-3p+4)\Longrightarrow d=\gcd(p+1,-5p+3)=\gcd(p+1,8)\iff d\mid 8$

Dễ thấy $\gcd(3,3^{n-1}+1)=1$ nên ta có các trường hợp sau:

$d=1\Longrightarrow \left\{\begin{matrix} p^2-3p+4=3^{n-1}+1\\ p+1=3 \end{matrix}\right.\iff (n,p)=(1,2)$

 

$d=2\Longrightarrow \left\{\begin{matrix} \frac{p^2-3p+4}{2}=\frac{3^{n-1}+1}{4}\\ \frac{p+1}{2}=3 \end{matrix}\right.\iff (n,p)=(4,5)$

 

$d=4\Longrightarrow \left\{\begin{matrix} \frac{p^2-3p+4}{4}=\frac{3^{n-1}+1}{16}\\ \frac{p+1}{4}=3 \end{matrix}\right.\iff PT$ vô nghiệm

 

$d=8\Longrightarrow \left\{\begin{matrix} \frac{p^2-3p+4}{8}=\frac{3^{n-1}+1}{64}\\ \frac{p+1}{8}=3 \end{matrix}\right.\iff PT$ vô nghiệm

 

Vậy $(n,p)=(1,2);(4,5)$




#653797 Trường hè Toán học Miền Nam.

Đã gửi bởi Minhnguyenthe333 on 11-09-2016 - 20:40 trong Thi HSG cấp Tỉnh, Thành phố. Olympic 30-4. Đề thi và kiểm tra đội tuyển các cấp.

5/

Từ điều kiện đề bài, ta thấy số cách chọn tối đa tuân theo quy luật dãy số Fibonacci:$\left\{\begin{matrix}F_1=1;F_2=2\\F_n=F_{n-1}+F_{n-2} \end{matrix}\right.$

$\implies$ Với $10$ người thì có tối đa $89$ cách sắp xếp theo chuẩn thứ tự




#601084 Trong mặt phẳng tọa độ Oxy cho ba điểm A(2;5), B(-1;-1) và C(4;9). Viết phươn...

Đã gửi bởi Minhnguyenthe333 on 01-12-2015 - 20:30 trong Phương pháp tọa độ trong mặt phẳng

Trong mặt phẳng tọa độ Oxy cho ba điểm A(2;5), B(-1;-1) và C(4;9).
a. Viết phương trình đường thẳng BC.
b. Chứng minh ba điểm A, B, C thẳng hàng.
c. Chứng minh rằng ba đường thẳng BC; $3x-y-1=0$ và $x-2y+8=0$ đồng quy.

a.Do $B,C$ thẳng hàng nên ta có hệ: $\left\{\begin{matrix} -1=-a+b & \\ 9=4a+b & \end{matrix}\right.$
Giải hệ này suy ra pt đường thẳng $BC: y=2x+1$
b.Thay toạ độ điểm $A$ vào pt đường thẳng $BC$ thoả mãn nên $A\in $ đt $BC$ (đpcm)
c.3 đt đồng quy nên tồn tại $x$ thoả 3 pt: $2x+1=3x-1=\frac{x+8}{2}\Leftrightarrow x=2,y=5$
Vậy 3 đường thẳng đồng quy tại $M(2;5)$ (đpcm)



#648865 Topic: [LTDH] Mỗi ngày hai bất đẳng thức.

Đã gửi bởi Minhnguyenthe333 on 10-08-2016 - 09:01 trong Bất đẳng thức và cực trị

Tiếp theo: 

Bài 7: Cho $a,b,c$ là các số thực dương thỏa mãn: $a^3+b^3+c^3=3$. Chứng minh rằng:

$\frac{a^3}{b^2-2b+3}+\frac{2b^3}{c^3+a^2-2a-3c+7}+\frac{3c^3}{a^4+b^4+a^2-2b^2-6a+11}\le \frac{3}{2}$

 

Ta xét mẫu số của 3 phân thức:

Dự đoán $a=b=c=1$ thì DBXR

Chú ý rằng: 

  $b^2-2b+3=(b-1)^2+2\geqslant 2$

  

  $c^3+a^2-2a-3c+7=(c-1)^2(c+2)+(a-1)^2+4\geqslant 4$

 

  $a^4+b^4+a^2-2b^2-6a+11=(a^2-1)^2+3(a-1)^2+(b^2-1)^2+6\geqslant 6$

 

$\Longrightarrow VT\leqslant \frac{a^3}{2}+\frac{2b^3}{4}+\frac{3c^3}{6}=\frac{a^3+b^3+c^3}{2}=\frac{3}{2}$

Dấu $"="$ xảy ra khi $a=b=c=1$




#648604 Topic: [LTDH] Mỗi ngày hai bất đẳng thức.

Đã gửi bởi Minhnguyenthe333 on 08-08-2016 - 18:45 trong Bất đẳng thức và cực trị

Bài 1: Cho $a,b,c$ là các số thực dương thỏa mãn: $abc=1$. Chứng minh: 
 $\frac{1}{a}+\frac{1}{b}+\frac{1}{c}+\frac{13}{a+b+c+1}\ge \frac{25}{4}$

Mình làm lại bài 1 như sau:

Đổi biến $(a,b,c)\rightarrow \left (\frac{x^2}{yz},\frac{y^2}{zx},\frac{z^2}{xy}\right )$
Giả sử $x\leqslant y\leqslant z\Longrightarrow z^2\geqslant xy$

BĐT$\iff \sum \frac{xy}{z^2}+\frac{13xyz}{x^3+y^3+z^3+xyz}\geqslant \frac{25}{4}$

$\iff \frac{x^3y^3+y^3z^3+z^3x^3-3x^2y^2z^2}{x^2y^2z^2}\geqslant \frac{13(x^3+y^3+z^3-3xyz)}{4(x^3+y^3+z^3+xyz)}$

Ta có: $x^3+y^3+z^3-3xyz=(x+y+z)\left [(x-y)^2+(z-x)(z-y)\right ]$
Và $z^2\geqslant xy\Longrightarrow x^3y^3+y^3z^3+z^3x^3-3x^2y^2z^2$
$=(xy+yz+zx)\left [z^2(x-y)^2+xy(z-x)(z-y)\right ]\geqslant xy(xy+yz+zx)\left [(x-y)^2+(z-x)(z-y)\right ]$

Do đó ta chỉ cần chứng minh:
$\frac{xy(xy+yz+zx)\left [(x-y)^2+(z-x)(z-y)\right ]}{x^2y^2z^2}\geqslant \frac{13(x+y+z)\left [(x-y)^2+(z-x)(z-y)\right ]}{4(x^3+y^3+z^3+xyz)}$

$\iff 4(xy+yz+zx)(x^3+y^3+z^3+xyz)\geqslant 13xyz^2(x+y+z)$

Do tính thuần nhất nên ta chuẩn hóa $x+y=2\Longrightarrow xy\leqslant 1\leqslant z$ và $x^3+y^3\geqslant 2$

$\iff 4(2z+xy)(2+z^3+xyz)\geqslant 13xyz^2(z+2)$
$\iff (8z^4-9z^3-18z^2+20z+8)+(1-xy)(9z^3+18z^2-4z-4xyz-8)\geqslant 0$
Chú ý rằng với $z\geqslant 1$, ta luôn có
$8z^4-9z^3-18z^2+20z+8>0$

$9z^3+18z^2-4z-4xyz-8\geqslant 9z^3+18z^2-4z-4z^3-8=5z^3+18z^2-4z-8>0 $

Vậy ta có đpcm, dấu "=" xảy ra khi $a=b=c=1$




#648743 Topic: [LTDH] Mỗi ngày hai bất đẳng thức.

Đã gửi bởi Minhnguyenthe333 on 09-08-2016 - 15:03 trong Bất đẳng thức và cực trị

Từ đó ta thu được $1\leqslant t< 2\Leftrightarrow \frac{1}{2}\leqslant xy< 1$

 

Mình có cách ngắn hơn cho phần này: 

Đặt $t=xy$

Từ giả thiết chú ý rằng: $\sqrt{2xy}\leqslant \frac{1}{2}+xy$

$\Longrightarrow x^4+y^4\leqslant 2-\frac{1}{xy}+xy\iff 2x^2y^2\leqslant 2-\frac{1}{xy}+xy$

$\iff 2t^3-t^2-2t+1\leqslant 0\iff (t+1)(t-1)(2t-1)\leqslant 0\iff \frac{1}{2}\leqslant t \leqslant 1$




#648448 Topic: [LTDH] Mỗi ngày hai bất đẳng thức.

Đã gửi bởi Minhnguyenthe333 on 07-08-2016 - 19:46 trong Bất đẳng thức và cực trị

Bài 1: Cho $a,b,c$ là các số thực dương thỏa mãn: $abc=1$. Chứng minh:
$\frac{1}{a}+\frac{1}{b}+\frac{1}{c}+\frac{13}{a+b+c+1}\ge \frac{25}{4}$

Chém bài 1 :)) :
Biến đổi $(a,b,c)\Rightarrow \left (\frac{x^2}{yz},\frac{y^2}{xz},\frac{z^2}{xy}\right )$

BĐT$\iff \frac{x^3+y^3+z^3}{xyz}+\frac{13xyz}{x^3+y^3+z^3+xyz}\geqslant \frac{25}{4}$

$\iff \frac{x^3+y^3+z^3+xyz}{xyz}+\frac{13xyz}{x^3+y^3+z^3+xyz}\geqslant \frac{29}{4}$

Ta đặt $t=\frac{x^3+y^3+z^3+xyz}{xyz}$
$\iff t+\frac{13}{t}\geqslant \frac{29}{4}\iff (t-4)(4t-13)\geqslant 0\iff t\geqslant 4$
Điều này luôn đúng vì $t\geqslant 4\iff x^3+y^3+z^3\geqslant 3xyz$
Dấu "=" xảy ra khi $a=b=c=1$



#648537 Topic: [LTDH] Mỗi ngày hai bất đẳng thức.

Đã gửi bởi Minhnguyenthe333 on 08-08-2016 - 11:11 trong Bất đẳng thức và cực trị

 

Bài 3: Cho $a,b,c$ là các số thực dương thỏa mãn: $a+b+c=3$. Chứng minh rằng: $3(a^2+b^2+c^2)+4abc\ge 13$.

Có thể mở rộng bài toán: Tìm hằng số $k$ lớn nhất để bất đẳng thức sau đúng:

                           $a^2+b^2+c^2+kabc\geqslant 3+k$

P/s: Chọn $k=\frac{4}{3}$ ta có bài toán trên




#648486 Topic: [LTDH] Mỗi ngày hai bất đẳng thức.

Đã gửi bởi Minhnguyenthe333 on 07-08-2016 - 22:50 trong Bất đẳng thức và cực trị

Chỗ màu đỏ bạn biến đổi như thế nào


Mình biến đổi sai rồi bạn ơi



#614974 Topic về phương trình và hệ phương trình

Đã gửi bởi Minhnguyenthe333 on 14-02-2016 - 17:10 trong Phương trình - hệ phương trình - bất phương trình

 

Đây là những bài tập chưa có lời giải trong Topic về phương trình và hệ phương trình, mong các bạn sớm hoàn thiện những bài tập này trước khi đăng bài mới để tránh loãng topic

 

 

Bài 221: $\left\{\begin{matrix} &(x+y)(x^{2}+y^{2})=15 \\ &(x-y)(x^{2}-y^{2})=3 \end{matrix}\right.$

Lấy $PT(1)$ chia $PT(2)<=>10xy=4(x^2+y^2)$

$<=>10t=4t^2+4$ ($t=\frac{x}{y}$)

$<=>(t-2)(2t-1)=0$.Đến đây suy ra $x=2y$ hay $y=2x$




#612715 Topic về phương trình và hệ phương trình

Đã gửi bởi Minhnguyenthe333 on 03-02-2016 - 18:02 trong Phương trình - hệ phương trình - bất phương trình

Thứ nhất: sau khi mình check thì đề có chút nhầm lẫn:

Lại là:

Bài 176: Tìm $x\in \left [ 0;3 \right ]$ thỏa mãn:

$x\sqrt{5-x}+\left ( 3-x \right )\sqrt{2+x}=\frac{3\sqrt{7}}{\sqrt{2}}$

Thứ hai: Lời giải của bác cũng có nhầm lẫn:

$18-6x(x-3)<(\frac{63}{2})^2 (?????)$ lưu ý điều kiện $x\in \left [ 0;3 \right ]$

P/S: $x=\frac{3}{2}$ là Max xảy ra

P/S lần nữa: Mời mọi người! Xin hết ~!

$VT^2=18-6x(x-3)\leqslant \frac{63}{2}<=>-(2x-3)^2\leqslant 0$ (luôn đúng)
Dấu "=" xảy ra nên $x=\frac{3}{2}$



#612312 Topic về phương trình và hệ phương trình

Đã gửi bởi Minhnguyenthe333 on 01-02-2016 - 20:30 trong Phương trình - hệ phương trình - bất phương trình

Bài 175: Tìm nghiệm nguyên không âm của hệ phương trình

             $\left\{\begin{matrix}2^x-|y^2-y|=1\\2^x+|y-1|<1 \end{matrix}\right.$




#612296 Topic về phương trình và hệ phương trình

Đã gửi bởi Minhnguyenthe333 on 01-02-2016 - 19:35 trong Phương trình - hệ phương trình - bất phương trình

Bài 171: Giải phương trình (bài khá hay :)) )
$\frac{1001x^4+x^4\sqrt{2x^2+2002}+4x^2}{999}=2002$

Bài 172: Giải phương trình: $\sqrt{2x+1}+\sqrt{17-2x}=x^4-8x^3+17x^2-8x+22$



#616124 Topic về phương trình và hệ phương trình

Đã gửi bởi Minhnguyenthe333 on 20-02-2016 - 20:14 trong Phương trình - hệ phương trình - bất phương trình

Bài 253:Giải hệ phương trình $(n$ là số tự nhiên và $n\geqslant 2)$

     $\left\{\begin{matrix}x_1^2=x_2+1\\x_2^2=x_3+1\\.................. \\ x_{n-1}^2=x_n+1\\x_n^2=x_1+1\end{matrix}\right.$




#612522 Topic về phương trình và hệ phương trình

Đã gửi bởi Minhnguyenthe333 on 02-02-2016 - 20:53 trong Phương trình - hệ phương trình - bất phương trình

Bài 'chế' từ BĐT của phương trình trên là min còn một phương trình nữa là max như sau.

Bài 176: Tìm $x\in \left [ 0;3 \right ]$ thỏa mãn:

$x\sqrt{5-x}+\left ( 3-x \right )\sqrt{2+x}=\frac{63}{2}$

Tương tự thì $VT^2=18+x(x-3)+2x(3-x)\sqrt{(5-x)(x+2)}\leqslant 18+x(x-3)+7x(3-x)=18-6x(x-3)<(\frac{63}{2})^2$
=>PT vô nghiệm



#617144 Topic về phương trình và hệ phương trình

Đã gửi bởi Minhnguyenthe333 on 26-02-2016 - 22:35 trong Phương trình - hệ phương trình - bất phương trình

Bài 282: Tìm $a$ để hệ sau có nghiệm duy nhất:

       $\left\{\begin{matrix} x^2+|a+1|x\leqslant x^5-7x^2+x+2\\ x^4+x^3+(a^2-3)x^2=4x+4+4a^2\end{matrix}\right.$




#612965 Topic về phương trình và hệ phương trình

Đã gửi bởi Minhnguyenthe333 on 05-02-2016 - 09:35 trong Phương trình - hệ phương trình - bất phương trình

Giải phương trình $\sqrt{\frac{1-x}{x}}=\frac{2x+x^2}{1+x^2}$

ĐKXĐ: $\frac{1}{2}\leqslant x\leqslant 1$
Ta có đánh giá sau:
$VT=1.\sqrt{\frac{1}{x}-1}\leqslant \frac{1}{2x}$ (theo AM-GM)
Mặt khác: $VP- \frac{1}{2x}=\frac{2x^3+3x^2-1}{2x(x^2+1)}=\frac{(x+1)^2(2x-1)}{2x(x^2+1)}\geqslant 0$ (luôn đúng)
Dấu "=" xảy ra nên. $x=\frac{1}{2}$



#613644 Topic về phương trình và hệ phương trình

Đã gửi bởi Minhnguyenthe333 on 08-02-2016 - 14:35 trong Phương trình - hệ phương trình - bất phương trình

Bài 203: Giải phương trình:

                $\frac{|x|\sqrt{x^2+1}-x^2-3+2\sqrt{2}}{|x|\sqrt{x^2+1}+x^2+3-2\sqrt{2}}=x^2$